2022 AMC 10B Problems/Problem 24

Revision as of 07:10, 4 December 2022 by MRENTHUSIASM (talk | contribs)

Problem

Consider functions $f$ that satisfy \[|f(x)-f(y)|\leq \frac{1}{2}|x-y|\] for all real numbers $x$ and $y$. Of all such functions that also satisfy the equation $f(300) = f(900)$, what is the greatest possible value of \[f(f(800))-f(f(400))?\] $\textbf{(A)}\ 25 \qquad\textbf{(B)}\ 50 \qquad\textbf{(C)}\ 100 \qquad\textbf{(D)}\ 150 \qquad\textbf{(E)}\ 200$

Solution 1 (Absolute Values and Inequalities)

By definition, we have \begin{align*} |f(f(800))-f(f(400))| &\leq \frac12|f(800)-f(400)| &&(\bigstar) \\ &\leq \frac12\left|\frac12|800-400|\right| \\ &= 100, \end{align*} from which we eliminate answer choices $\textbf{(D)}$ and \textbf{(E)}.$Note that <cmath>\begin{align*} |f(800)-f(300)|&\leq 250, \\ |f(800)-f(900)|&\leq 50, \\ |f(400)-f(300)|&\leq 50, \\ |f(400)-f(900)|&\leq 250. \\ \end{align*}</cmath> Let$ (Error compiling LaTeX. Unknown error_msg)a=f(300)=f(900).$Together, we conclude that <cmath>\begin{align*} |f(800)-a|&\leq 50, \\ |f(400)-a|&\leq 50. \\ \end{align*}</cmath> We rewrite$ (Error compiling LaTeX. Unknown error_msg)(\bigstar)$as <cmath>\begin{align*} |f(f(800))-f(f(400))| &\leq \frac12|f(800)-f(400)| \\ &= \frac12|(f(800)-a)-(f(400)-a)| \\ &\leq \frac12|50-(-50)| \\ &=\boxed{\textbf{(B)}\ 50}. \end{align*}</cmath> ~MRENTHUSIASM

==Solution 2 (Lipschitz Condition)==

Denote$ (Error compiling LaTeX. Unknown error_msg)f(900)-f(600) = a$. Because$f(300) = f(900)$,$f(300) - f(600) = a$.

Following from the Lipschitz condition given in this problem,$ (Error compiling LaTeX. Unknown error_msg)|a| \leq 150$and <cmath> \[ f(800) - f(600) \leq \min \left\{ a + 50 , 100 \right\} \] </cmath> and <cmath> \[ f(400) - f(600) \geq \max \left\{ a - 50 , -100 \right\} . \] </cmath> Thus, <cmath> \begin{align*} f(800) - f(400) & \leq \min \left\{ a + 50 , 100 \right\} - \max \left\{ a - 50 , -100 \right\} \\ & = 100 + \min \left\{ a, 50 \right\} - \max \left\{ a , - 50 \right\} \\ & = 100 + \left\{ \begin{array}{ll} a + 50 & \mbox{ if } a \leq -50 \\ 0 & \mbox{ if } -50 < a < 50 \\ -a + 50 & \mbox{ if } a \geq 50 \end{array} \right. . \end{align*} </cmath> Thus,$ (Error compiling LaTeX. Unknown error_msg)f(800) - f(400)$is maximized at$a = 0$,$f(800)-f(600) = 50$,$f(400)-f(600)=-50$, with the maximal value 100.

By symmetry, following from an analogous argument, we can show that$ (Error compiling LaTeX. Unknown error_msg)f(800) - f(400)$is minimized at$a = 0$,$f(800)-f(600) = -50$,$f(400)-f(600)=50$, with the minimal value$-100$.

Following from the Lipschitz condition, <cmath> \begin{align*} f(f(800)) - f(f(400)) & \leq \frac{1}{2} \left| f(800) - f(400) \right| \\ & \leq 50 . \end{align*} </cmath> We have already construct instances in which the second inequality above is augmented to an equality.

Now, we construct an instance in which the first inequality above is augmented to an equality.

Consider the following piecewise-linear function: <cmath> \[ f(x) = \left\{ \begin{array}{ll} \frac{1}{2} \left( x - 300 \right) & \mbox{ if } x \leq 300 \\ -\frac{1}{2} \left( x - 300 \right) & \mbox{ if } 300 < x \leq 400 \\ \frac{1}{2} \left( x - 600 \right) & \mbox{ if } 400 < x \leq 800 \\ -\frac{1}{2} \left( x - 900 \right) & \mbox{ if } x > 800 \end{array} \right.. \] </cmath> Therefore, the maximum value of$ (Error compiling LaTeX. Unknown error_msg)f(f(800)) - f(f(400))$is$\boxed{\textbf{(B)}\ 50}$.

~Steven Chen (Professor Chen Education Palace, www.professorchenedu.com)

~Viliciri (LaTeX edits)

==Solution 3 (Educated Guess)==

Divide both sides by$ (Error compiling LaTeX. Unknown error_msg)|x - y|$to get$\frac{|f(x) - f(y)|}{|x - y|} \leq \frac{1}{2}$. This means that when we take any two points on$f$, the absolute value of the slope between the two points is at most$\frac{1}{2}$.

Let$ (Error compiling LaTeX. Unknown error_msg)f(300) = f(900) = c$, and since we want to find the maximum value of$|f(800) - f(400)|$, we can take the most extreme case and draw a line with slope$\frac{-1}{2}$down from$f(300)$to$f(400)$and a line with slope$\frac{-1}{2}$up from$f(900)$to$f(800)$. Then$f(400) = c - 50$and$f(800) = c + 50$, so$|f(800) - f(400)| = |c + 50 - (c - 50)| = 100$, and this is attainable because the slope of the line connecting$f(400)$and$f(800)$still has absolute value less than$\frac{1}{2}$.

Therefore,$ (Error compiling LaTeX. Unknown error_msg)|f(f(800)) - f(f(400))| \leq \frac{1}{2}|f(800) - f(400)| = \frac{1}{2}(100) = \boxed{\textbf{(B)}\ 50}$.

Video Solution

https://youtu.be/2Li0IYOQCFQ

~Steven Chen (Professor Chen Education Palace, www.professorchenedu.com)

Video Solution by OmegaLearn Using Algebraic Manipulation

https://youtu.be/-yzpw6b3_KA

~ pi_is_3.14

See Also

2022 AMC 10B (ProblemsAnswer KeyResources)
Preceded by
Problem 23
Followed by
Problem 25
1 2 3 4 5 6 7 8 9 10 11 12 13 14 15 16 17 18 19 20 21 22 23 24 25
All AMC 10 Problems and Solutions

The problems on this page are copyrighted by the Mathematical Association of America's American Mathematics Competitions. AMC logo.png